Download as pdf
Download as pdf
You are on page 1of 22
Chapter - 6 yosPITAL’S RULE fn introduction , rieinfine ints donor always obey the four fundamental laws of arithimetté, ‘his means that the sum OF product of the limits need not be esual 10 the limit of the sum or ped 0 hus, « method which enables ts €0 evaluate most of the limits in such exceptional cases is known jl! Hopital 's Rule or L Hospital’s Rule, after the French Mathematician Guillaume Francois Antoine ce Hopital 62: Indeterminate forms {oHospital’s Rule ( theorem) allows the evaluation of indeterminate form Oe OS 2-o, Oxm, 1°, wo", 0. 0 0 (i) —form: ) an iat h) lim h d | i +h) beh) -1 _ f Ys fit Lb tiny lim lex tim dh a "1h "nt Dah “(h) ah k + cose Problem — 7; Find 3 values of and / such that lin 4, Slaton : The denominator of the netion tends to zor as i» The given traction will tend wy rh init i and only if te meme NO also tends to zor ag y +9 This requires k-+ | = k (1) Fok =~ 1, Wehnve K+ cosir/9 fim form a= (Applying [> de (Paying L Hopital fu) Moose | Pri 1 = So} Given, ~1 ps, #3)... 4 1 Probie ‘ f mee 28) ese, Why | Homitas ite de ‘ sin/ d} ' “NOt Apply the prow, ad (0) Find the timt Problem linn — Pro tang Solution : (a) jo mtn 2 (0) Lot Ry) = sim 8) = gin F(x) inthe © (x) im the © fey 0 r(x) i format x A & jopital’s rule is not applicable, 4 ean!) in ts —* “cated (sw) " put Hs I 1 2xsin— cos cose i 1 1) then fim| 2 sin-” —c08-" | would exists and consequently 1 1 tim &os— would axis | fy een 0], x I which is not true [ cos—oscillates between — J and las x +] 2? sin Wehave 0.<|—_ sinx | sine] |sinx] e 1, therefore lim — 0 eo sine » by squeezing rule. — if it exists x sin , lim sin —. which does not exist. ave sing ort [Ey cee oe (sin2x a Find the constants a and b so that lim) = Soon: Lett. = tim{ SH 2t 0) = gaat arb? eum ar mt = 2oos2x+a+3be = in - S re (L'Hopital’s raley imi is ofthe indeterminate for (The above I “ 6x ' lim ee : (LHopital’s rate) 6b=8 é Bu L = a “cept possibilty # Case—3. Hence, tim ez) Problem ~ 12. Saletion.. lin = in br (= form) Fak form) form) abt + (L’ Hopital’s rule) fin og + es fie ay Hence to find this limit, We Pr 6.5 : Indeterminate form co — co By some algebraic or trigonome 0 « one of the two standard forth ao Problem 15: Evaiiate titn | Slaton’ The given lit has the form lim —* lim — = fim | —* x | aceed as follows: sinx we Process, we can transform indeterminate form, OF this type za” Then, we can use L’ Hopital 's rule 0 evalute its lin I teey n 2 — 00; because : 11> ** and wa e ys so 66; indaterminate form 0. © Problem — 18: Solution Probiem — yy Solution Problem — 39 : Solution ort fan he ICtion, 4, im oe (form) , 6x ar im ——— oieo,O1) xr (0.01) e' 6x fit ——porr =? MS (oly e y=0 (x axis) is the horizontal asymptote. i 1 | prtien=21 Evaluate HM 3°) sin| — | ot | | J | Solution ; im x°= +00 1 1 1 and fim =0 ; x) x 6x | The given limit has the form 0.<« “{sa( 2 ] siny~y+t yy = lim ge (oY Sf tom) (+= ys 00s 240 ) 0 x 1 cos y-14— 3 tim y * ——— $__(1Hopital’s rule) 7 lin =Siny+99(0, _) oT 3 form ; oy a ™) - lim =208) +1 60,° (’Hopital’s rule) , Bn MD... ri 1205 (L’Hopital’s rule) “ lim £08 1 120 ~ 130 8, \ . tate forms of the type 0°, "1" and g be two real valued functions defined in a neighbourhood of a, except possibly atx =a WDpose:F ¢e) paws: fe) (for all © in the neighbourhood of a. Hinen Inv ayy = 2) In PO) snd feo 19) der to evalt ynction, Henwe i cxpunsetial Vinton 0 « corinne a, then (fly) is any one off the forme: OF, a te Yeh fla). He then gt} le Ing he hve ses, ew ts ella Fierine cf this fox fit 7 ruben 12.5) Hienluiie Hier! * (1 fear) Solution: Let k= Sin io ee ly | , Problem — 295 Pind tin | 1% ab ; itm ini ia ¢ «} y ; Inj ia @) bh, tiny ae 7 te — : , ‘ _ | ; [le } E b lim s | i b lim {2x | : leg) "ba | | Iny ab . Te Vo On 4 ‘ ‘i a . 4 tin (142) x ins an ical sym tthe graph the fin, pane 3)" po) using L’Hopital’s rule “toe io the tte fore 1°, when x > 4a soutiot? fret bial asymptotes, we compute eal x+2 ows a~ fim (x43 tn tim | = inl win?) [L’Hopitat’s rule} = 2hm ————_ aa = em) = Zit —2 wm yannaes = 2im sm Ox+5 =e Thus y= e° is a horizontal asymptote. Problem — 25 : Find Aso that tim (224) =§ a 2A, ‘Solution: Li x+Aa \ we =) isin 1° form 1. Show that lim ¥+SINX _ 1 and show that L’Hopital’s rule cannot t m2 x +C0SX 2. Evaluate the following limits. 4 Show that L’Hopital’s rule is not applicable in the following cases x —~cosx (a) lim =e (b) x x (c) lim |sin.x} +| cos.x| (a) | X SINX + Cos x 90 = xe 3. Evaluate the following limits: F tan? x— x’ [ (a) lin— (b) lim x? In| bi eis x 0 x? tan? x

You might also like